Difference between revisions of "2022 AMC 10B Problems/Problem 21"
Numerophile (talk | contribs) (→Problem) |
Mathfan2020 (talk | contribs) (Added Solution 2) |
||
Line 28: | Line 28: | ||
<cmath>111(a - 11) + 1233 = 101(b - 12) + 1233 = P(x)</cmath>. | <cmath>111(a - 11) + 1233 = 101(b - 12) + 1233 = P(x)</cmath>. | ||
− | Therefore <math>a = 11_{x} = x + 1</math>, <math>b = 12_{x} = x + 2</math>, and <math>P(x) = 1233_{x} = x^3 + 2x^2 + 3x + 3</math>. <math>3</math> is the minimal degree of <math>P(x)</math> since there is no way to influence the <math>x</math>‘s digit in <math>101b + 21</math> when <math>b</math> is an integer. The desired sum is <math>1^2 + 2^2 + 3^2 + 3^2 = \boxed{\textbf{(E) 23 | + | Therefore <math>a = 11_{x} = x + 1</math>, <math>b = 12_{x} = x + 2</math>, and <math>P(x) = 1233_{x} = x^3 + 2x^2 + 3x + 3</math>. <math>3</math> is the minimal degree of <math>P(x)</math> since there is no way to influence the <math>x</math>‘s digit in <math>101b + 21</math> when <math>b</math> is an integer. The desired sum is <math>1^2 + 2^2 + 3^2 + 3^2 = \boxed{\textbf{(E)} \ 23}</math> |
P.S. The 4 computational steps can be deduced through quick experimentation. | P.S. The 4 computational steps can be deduced through quick experimentation. | ||
~ numerophile | ~ numerophile | ||
+ | |||
+ | ==Solution 2== | ||
+ | |||
+ | Let <math>P(x) = Q(x)(x^2+x+1) + x + 2</math>, then <math>P(x) = Q(x)(x^2+1) + xQ(x) + x + 2</math>, therefore <math>xQ(x) + x + 2 \equiv 2x + 1 \pmod{x^2+1}</math>, or <math>xQ(x) \equiv x-1 \pmod{x^2+1}</math>. Clearly the minimum is when <math>Q(x) = x-1</math>, and expanding gives <math>P(x) = x^3+2x^2+3x+3</math>. Summing the squares of coefficients gives <math>\boxed{\textbf{(E)} \ 23}</math> | ||
+ | |||
+ | ~mathfan2020 | ||
==Video Solutions== | ==Video Solutions== |
Revision as of 21:35, 18 November 2022
Problem
Let be a polynomial with rational coefficients such that when is divided by the polynomial , the remainder is , and when is divided by the polynomial , the remainder is . There is a unique polynomial of least degree with these two properties. What is the sum of the squares of the coefficients of that polynomial?
Solution 1 (Experimentation)
Given that all the answer choices and coefficients are integers, we hope that has positive integer coefficients.
Throughout this solution, we will express all polynomials in base . E.g. .
We are given:
.
We add and to each side and balance respectively:
We make the units digits equal:
We now notice that:
.
Therefore , , and . is the minimal degree of since there is no way to influence the ‘s digit in when is an integer. The desired sum is
P.S. The 4 computational steps can be deduced through quick experimentation.
~ numerophile
Solution 2
Let , then , therefore , or . Clearly the minimum is when , and expanding gives . Summing the squares of coefficients gives
~mathfan2020
Video Solutions
~ ThePuzzlr
~Steven Chen (Professor Chen Education Palace, www.professorchenedu.com)
See Also
2022 AMC 10B (Problems • Answer Key • Resources) | ||
Preceded by Problem 20 |
Followed by Problem 22 | |
1 • 2 • 3 • 4 • 5 • 6 • 7 • 8 • 9 • 10 • 11 • 12 • 13 • 14 • 15 • 16 • 17 • 18 • 19 • 20 • 21 • 22 • 23 • 24 • 25 | ||
All AMC 10 Problems and Solutions |
The problems on this page are copyrighted by the Mathematical Association of America's American Mathematics Competitions.